wayne_palmer10
Thanks Received: 0
Forum Guests
 
Posts: 17
Joined: July 04th, 2009
 
 
 

Q14 - If the proposed tax reduction

by wayne_palmer10 Wed Sep 02, 2009 11:47 am

This is a sufficient assumption question. I tried diagramming the stimulus in order to find the missing premise but had some difficulty.
User avatar
 
ManhattanPrepLSAT2
Thanks Received: 311
Atticus Finch
Atticus Finch
 
Posts: 303
Joined: July 14th, 2009
 
 
 

Re: Q14 - If the proposed tax reduction

by ManhattanPrepLSAT2 Tue Sep 15, 2009 3:15 pm

The argument contains a series of conditional premises that can be considered truths:

If tax reduction adopted -- > library will be forced to discontinue.

If library forced to discontinue -- > parents inconvenienced.

We can combine this conditionals to conclude:

If tax reduction adopted -- > parents will be inconvenienced.

Then the argument reaches a conclusion that, because of these conditional truths, the tax reduction package will not be adopted this year.

Notice, there is a big gap between the premises and the conclusion. Just because the parents will be inconvenienced does not mean, necessarily, that the tax plan won't be adopted.

In order for the conclusion to be true, we need to know that the relationship works in the other direction -- we need to know that parents being inconvenienced does indeed make it so that the plan won't be adopted. Answer choice (D) gives us exactly what we need.

If we write (D) in conditional terms, it would read:

"If plan greatly inconveniences parents, will not be adopted."

If D is assumed to be true, and inserted into the original argument, it would make the conclusion sound. Since we know the plan would greatly inconvenience parents, we know it will not be adopted.
 
rsmorale
Thanks Received: 3
Jackie Chiles
Jackie Chiles
 
Posts: 37
Joined: February 20th, 2011
 
 
trophy
Most Thankful
 

Re: Q14 - If the proposed tax reduction

by rsmorale Tue Jul 26, 2011 5:10 pm

How do you diagram D?
User avatar
 
ManhattanPrepLSAT2
Thanks Received: 311
Atticus Finch
Atticus Finch
 
Posts: 303
Joined: July 14th, 2009
 
This post thanked 1 time.
 
 

Re: Q14 - If the proposed tax reduction

by ManhattanPrepLSAT2 Thu Aug 04, 2011 11:12 am

Remember that in the original argument we had:

If tax reduction adopted --> Parents will be inconvenienced.

You could diagram (D):

Tax reduction package inconveniences parents --> will NOT be adopted this year.

If we add this to the chain we already have, we can validly conclude that the tax reduction will not be adopted this year.

Hope that helps.
 
goriano
Thanks Received: 12
Atticus Finch
Atticus Finch
 
Posts: 113
Joined: December 03rd, 2011
 
 
 

Re: Q14 - If the proposed tax reduction

by goriano Sun May 06, 2012 7:21 pm

Mike.Kim Wrote:Remember that in the original argument we had:

If tax reduction adopted --> Parents will be inconvenienced.

You could diagram (D):

Tax reduction package inconveniences parents --> will NOT be adopted this year.

If we add this to the chain we already have, we can validly conclude that the tax reduction will not be adopted this year.

Hope that helps.


Could you talk about how to differentiate between (D) and (E)? I knew that I was looking for the contrapositive of the logic chain, specifically establishing that "many parents would NOT be greatly inconvenienced." Doesn't (E) say that as well?
 
timmydoeslsat
Thanks Received: 887
Atticus Finch
Atticus Finch
 
Posts: 1136
Joined: June 20th, 2011
 
This post thanked 1 time.
 
trophy
Most Thanked
trophy
First Responder
 

Re: Q14 - If the proposed tax reduction

by timmydoeslsat Sun May 06, 2012 9:10 pm

Excellent question.

The argument:

A ---> D ---> I
__________
~A

I can have a sufficient assumption of either ~D or ~I.

I expect the test writers to give me ~I due to the fact that if they did not, that would be a wasted premise.

The problem with answer choice E is that it is not forbidding me the idea that an inconvenience bill can be passed too. Perhaps both types are passed this year!
 
shaynfernandez
Thanks Received: 5
Elle Woods
Elle Woods
 
Posts: 91
Joined: July 14th, 2011
 
 
 

Re: Q14 - If the proposed tax reduction

by shaynfernandez Thu Jul 05, 2012 12:17 am

I think I am missing something. The argument is like this in conditional diagram form:
Tax adopted --> discontinue daily story hour
Discontinue daily story hour --> many parents inconvenienced

So we could create a chain:
Tax adopted --> discontinue DSH --> many parents inconv

The author concludes by saying: so, ~tax adopted

I know this is an sufficient assumption question but what would keep us from using this chain?

To get ~tax adopted couldn't we just take the contrapositive of our original chained statement?

~many parents inconv--> ~discont DSH --> ~tax adopted

I originally got this correct by not diagramming the conditional logic but strictly looking at what I perceived as the core. (tax reduction won't be implemented because the parents are going to be inconvenienced by the lack of a story hour)

It was pretty plain to see from that prospective that if parents are inconvenienced then the tax reduction will not be implemented.

How do we know that using the conditional chain doesn't yield an assumption ~many parents inconv --> ~tax adopted ?
 
goriano
Thanks Received: 12
Atticus Finch
Atticus Finch
 
Posts: 113
Joined: December 03rd, 2011
 
 
 

Re: Q14 - If the proposed tax reduction

by goriano Fri Jul 06, 2012 6:29 pm

timmydoeslsat Wrote:Excellent question.

The argument:

A ---> D ---> I
__________
~A

I can have a sufficient assumption of either ~D or ~I.

I expect the test writers to give me ~I due to the fact that if they did not, that would be a wasted premise.

The problem with answer choice E is that it is not forbidding me the idea that an inconvenience bill can be passed too. Perhaps both types are passed this year!


Timmy, I think I thought I understood your post when I first read it but now I don't understand anymore!

I get that the correct answer choice will establish ~D or ~I.

(D) says I --> not adopted

(E) says ~I --> adopted

Why does it matter that an inconvenience bill can be passed too? All we care about is establishing that something that WON'T inconvenience parents occurs. And (E) seems to say just that!
 
anjelica.grace
Thanks Received: 5
Jackie Chiles
Jackie Chiles
 
Posts: 41
Joined: November 17th, 2011
 
 
 

Re: Q14 - If the proposed tax reduction

by anjelica.grace Sun Aug 19, 2012 3:26 pm

timmydoeslsat Wrote:How do we know that using the conditional chain doesn't yield an assumption ~many parents inconv --> ~tax adopted ?


The conditional chain could yield the assumption we're looking for if it were established that this particular tax plan did not inconvenience parents, but we are told that it does.

Diagrammed, it looks like this:

~parents inconvenienced --> ~tax adopted. (contrapositive of chain)

parents inconvenienced. (the stimulus tells us this)

thus, we cannot infer ~tax adopted from the chain because the conditions don't match.

goriano Wrote:Why does it matter that an inconvenience bill can be passed too? All we care about is establishing that something that WON'T inconvenience parents occurs. And (E) seems to say just that!


The reason that we care whether an inconvenience bill can be passed is because that leaves the conclusion completed unjustified. If the arguments claims that this bill, which inconveniences parents, cannot be passed, the fact than an inconvenience bill can be passed directly goes against the conclusion. We don't want that for a sufficient question.

(E) doesn't exactly establish that something won't inconvenience parents occurs as you claim. Rather, it's conditional.

Timmy is right in that ~D or ~I in an answer choice would sufficiently justify the conclusion.

But as you pointed out, (E) says ~I --> A. That's conditional in nature, not absolute like we want.

If we still set up the conditional chain from the stimulus, it would be:

A --> D --> I.
∴ ~A

As we can all agree ~I or ~D would do the job.
A --> D --> I.
~I.
∴ ~A.

But that's not what we get in (E), which is ~I -->A.

A --> D --> I.
~I --> A.
∴ ~A

See the difference.

Further, notice that while the correct answer (D), which says I --> ~A, is not absolute but conditional in nature, it perfectly links the evidence and the conclusion.

A --> D --> I. (In other words, this bill will inconvenience parents.)
I --> ~A. (Anything that inconveniences parents will not be adopted.)
∴ ~A (Therefore, this bill will not be adopted.)

I sincerely hope this helped more than hurt anyone's understanding of the question.
 
josh.glenn44
Thanks Received: 0
Forum Guests
 
Posts: 7
Joined: October 13th, 2012
 
 
 

Re: Q14 - If the proposed tax reduction

by josh.glenn44 Wed Oct 24, 2012 9:03 am

Ok, I got this part:

A --> D --> I.
∴ ~A

Now when you diagram Answer Choice D, you get:

I--> ~A

(I see that from this we can determine ~I---> ~D --> ~A)

But where my confusion is how can I reason just to use the ~I??
I mean, if I use the full diagram from Answer Choice D, I've got:

A--> ~I --> ~D---> ~A

and then the contrapositive:

A--> D--> I --> ~A

and these two chains just don't make any sense (A--> ~A / ~A --> A)

So, the big question I have is, how do I know that I should just use ~I from Answer Choice D??
 
timmydoeslsat
Thanks Received: 887
Atticus Finch
Atticus Finch
 
Posts: 1136
Joined: June 20th, 2011
 
 
trophy
Most Thanked
trophy
First Responder
 

Re: Q14 - If the proposed tax reduction

by timmydoeslsat Wed Oct 24, 2012 6:48 pm

Good question. When we are diagramming these arguments for sufficient assumption questions, we are trying to get across the main ideas without losing important meaning.

So when we say: A ---> D ---> I ....

We are jotting down A to represent a proposed tax reduction plan being adopted. We know that this plan would lead to both D and I.

So we could represent that this is a specific plan of adoption by saying this:

A(x) ---> D ---> I

Answer choice D is telling us that a proposed tax reduction plan that is adopted will lead us to ~I.

A ---> D ---> I
__________
~A

So if we plug in this assumption (in bold), we can see that this will allow us to conclude ~A(x):

A(x) ---> D ---> I

A ---> ~I
__________
~A(x)

It is telling us that every single A must lead to ~I. Since this one subset of A leads to I, we know that this subset of A, A(x), will not take place.

The posts above are really just quick short hands for getting through the problem in an effective way.

We are told A ---> D ---> I .... We conclude with ~A, we are hunting for something about how we cannot have D or cannot have I.
 
griswald
Thanks Received: 0
Forum Guests
 
Posts: 6
Joined: August 16th, 2012
 
 
 

Re: Q14 - If the proposed tax reduction

by griswald Tue Mar 03, 2015 12:18 am

I quickly narrowed the questions down to D and E. I knew in my gut that it was D, but I wasted far too much time flipping between the two for some reason. How would you diagram E and why doesn't it also work as an answer?
 
asafezrati
Thanks Received: 6
Atticus Finch
Atticus Finch
 
Posts: 116
Joined: December 07th, 2014
 
 
 

Re: Q14 - If the proposed tax reduction

by asafezrati Mon Jun 01, 2015 4:24 pm

griswald Wrote:I quickly narrowed the questions down to D and E. I knew in my gut that it was D, but I wasted far too much time flipping between the two for some reason. How would you diagram E and why doesn't it also work as an answer?

It's actually a simple idea of conditionals which is very easy to learn.

E says something like this: Not Inconvenienced -> Will be Adopted.
The premises in the stimulus give us "Inconvinienced". The above rule doesn't allow you to get from this to "Will Not Adopted".

The contrapositive is Not Adopted -> Inconvenienced, and doesn't help us either, because, again, you need to get from the point in which parents are inconvenienced to the tax plan not being adopted. The contrapositive stated above only allow us to get to the point in which the parents are inconvenienced. Since we get this fact in the premise this rule is obsulete, it doesn't help us.

Is it clear?
 
M.M.
Thanks Received: 0
Vinny Gambini
Vinny Gambini
 
Posts: 9
Joined: September 02nd, 2012
 
 
 

Re: Q14 - If the proposed tax reduction

by M.M. Thu Aug 09, 2018 2:29 pm

I picked D but mostly due to the repetitive nature of the LSAT; I don't get why A is not a sufficient assumption though. If tax reduction package that will not force the library to discontinue daily story hours -> will be adopted this year as A states, isn't this the contrapositive of "proposed tax reduction package is adopted this year, the library will be forced to discontinue story hours" ? And thus wouldn't the conclusion you could draw from A be the conclusion in the argument? Is it because tax packages aren't necessarily mutually exclusive? Or is it just because D fits the argument better - i.e. it fits better into the "gaps" of the argument and thus allows it to be "more properly drawn" instead of just making the conclusion right?
User avatar
 
ohthatpatrick
Thanks Received: 3808
Atticus Finch
Atticus Finch
 
Posts: 4661
Joined: April 01st, 2011
 
 
 

Re: Q14 - If the proposed tax reduction

by ohthatpatrick Thu Aug 09, 2018 5:16 pm

There's nothing comparative about the question stem. It's not a RELATIVE matter of "which answer does the BEST job".

It's an ABSOLUTE matter of "Does this answer allow us to derive the conclusion, or not?"

On Sufficient Assumption, the four wrong answers are always wrong for the same reason: they don't allow us to derive the conclusion.

We're trying to derive the idea that:
proposed package will not be adopted this year

Here are the ingredients we have from the evidence:
adopt package --> discontinue daily story hours
and
discontinue daily story hours -> many parents inconvenienced

Since those have a common ingredient, we check to see if they can chain together, and indeed they can:
adopt package -> discontinue daily story hours -> many parents inconvenienced

the contrapositive of that chain looks like this:
NOT many parents inconvenienced -> daily story hours continued -> not adopt package

The contrapositive of the chain should seem more appealing to us because our goal is to prove:
not adopt package

(D) says, "this year, we will not inconvenience parents".

That triggers the chain we had and allows to derive "not adopt package".

Or you could hear it as
(D) "we won't adopt any package that inconveniences parents"
+ (from our evidence) "the proposed package would inconvenience parents"
------------------------------------------------------
Thus, we won't adopt the proposed package.

That's why (D) is correct.

------------------------

(A) says
"if the package allows daily story hours to continue, then it will be adopted".

Does the proposed package allow daily story hours to continue?

No, of course not. The proposed package will force us to discontinue daily story hours.
Since the rule in (A) doesn't apply to the proposed package we're talking about in the argument, it's worthless to us. It takes us nowhere else, since it doesn't apply to the proposed package we're discussing.

(A) would have been a correct answer had it said,
"Any tax reduction package that will force the library to discontinue daily story hours will not be adopted this year".

Sufficient Assumption is famous for writing trap answers that are just a negation or reversal of what we're actually looking for, so we have to make sure we can apply the answer choice we're picking to what we know from the premises, to see if it lets us derive the conclusion.

Hope this helps.
 
xiaot424
Thanks Received: 0
Vinny Gambini
Vinny Gambini
 
Posts: 1
Joined: January 15th, 2021
 
 
 

Re: Q14 - If the proposed tax reduction

by xiaot424 Fri Jan 15, 2021 7:58 am

Sparked by Timmy’s answer largely, together with all other great answers, I figured out the following notations make this question pretty EASY, especially when you are confused with choice D and E.

The logical chain in the argument :
A —> D —> I ( A for adopted; D for discontinue;I for inconvenienced )
——————-

~A


choice A: ~D —> A, add this to the chain in the argument, we get ~D —> A —> D —> I, obviously, we can not get ~A, wrong choice.

choice B:D —> I, just repeating part of the chain in the argument , we can not get ~A, wrong choice.

choice C:I —> D, add this to the chain in the argument, we get A —> D —> I —> D, still nothing about ~ A came up, wrong choice.

choice D: I —> ~A, add this to the chain in the argument, we get A —> D —> I —> ~A, at fist thought, it is just another logic chain, how can we get ~A? Here is the little twist, if you start with A , you end up in ~A; if you start with ~A, (LOL, this is just what we need). so, we can conclude ~A from this conditional chain. which is exactly what we need. right answer.

choice E : ~I —> A, add this to the chain in the argument, ~I —> A —> D —> I , still we can not get ~ A. wrong answer.


PS:(If we just change the last sentence in the argument to “so many parents will be greatly inconvenienced.” and if you know E is the right answer, then you have fully understood what I am talking about. )

PPS: You may ask, how do I know this question is about combining two conditional rules? The argument start with “If”, and scanning through the choices, they all start with no, any, every, and sort of conditional words. So be alert when those words poop up.

PPPS: ALL IN ALL, I think the real point the test writer wants us to know is using two conditional chains (one in the argument and one in the choice) to reach the conclusion/rather than what we commonly think that we need to find one element in the chain to reach the conclusion.

Hope this help! :P